Alegabraic
Consider the expression 2x²+y+4

1. How
many terms are there​

Answers

Answer 1

Answer:

3 terms is the answer of your question

Answer 2

this expression consists of 3 terms

Step-by-step explanation:

it is a quadratic equation in two

variables

with coefficient =4


Related Questions

express 7.36 as rational number​

Answers

Answer: The ratio would be 73.6

When the function f(x) = 4(2)x is changed to f(x) = 4(2)x − 13, what is the effect? (5 points)

a
There is no change to the graph because the exponential portion of the function remains the same.

b
The x-intercept is 13 spaces higher.

c
The y-intercept is 13 spaces lower.

d
All input values are moved 13 spaces to the left.

Answers

Answer: C. The y-intercept is 13 spaces lower.

Step-by-step explanation:

Answers in case you cant see them:

-3
-7
-11
11

Answers

Answer:

Step-by-step explanation:

8x-9+2+9x=180 degree(being co interior angles)

17x-7=180

17x=180+7

x=187/17

x=11 degree

Answer:

11

Step-by-step explanation:

consecutive interior angles always equal to 180 degrees, so you would add both equations together and equal it to 180 degrees. you would then simplify and solve.

8x-9 + 2+9x = 180

-7 + 17x = 180

-7+7     180+7 = 187

17x = 187

17x/17 = 187/17

x=11

Math challenging and now math challenging me to challenge math

Answers

Answer:

$30

Step-by-step explanation:

Total amount paid by the hire purchase scheme:

$60 + 6 * $45 = $60 + $270 = $330

Total amount paid in just the cash price:

$300

Difference:

$330 - $300 = $30

The expression -310 + 15m represents a submarine that began at a depth of 310 feet below sea level and ascended at a rate of 15 feet per minute. What was the depth of the submarine after 8 minutes?

Answers

Answer:

190 feet below sea level

Step-by-step explanation:

d = -310 + 15m

d = -310 + 15(8)

d = -310 + 120

d = -190

Help plz question in picture

Answers

Answer:

B and D are correct

Step-by-step explanation:

Imagine if c, b, and k were equal to 5.  Then, multiply to figure out how much Mia earned.  Solve the rest of the equations while still using 5 in place of the variables.  If they are the same as your first answer, it is right

Help me please I really need it

Answers

Answer:

2

Step-by-step explanation:

To find the average rate of change, we use:

[tex]\frac{changeofy}{changeofx} =rate ofchagne[/tex]

To find this lets look at the graph.

We see that at x=2, y=-3, and at x=3, y=-1.

The change of y is the first y coordinates minus the second, which in this case is:

-3-(-1)=-2

Change is the absolute value of the 1st and 2nd y. So its really a change of 2, not -2.

Our change of x falls under the same formula, being:

2-3=-1, but really being 1.

Now to plug this into our first formula:

2/1=2

So the rate of change is 2.

This seems to be your first answer.

Hope this helps!

Triangle A has sides 7, 12, and 18. Triangle B has sides 5, 9, and 5. Triangle C has sides 8, 8, and 8. What kind is each triangle? *

1. A: Scalene, B: Equilateral, C: Isosceles
2. A: Scalene, B: Isosceles, C: Equilateral
3. A: Isosceles, B: Scalene, C: Equilateral
4. A: Isosceles, B: Equilateral, C: Scalene

Answers

the answer is 2. because all sides of triangle c are equal and two side of triangle b are equal

Write the equation of the graph below.

Answers

Answer:

-x²+8x-15

Step-by-step explanation:

There are two solutions: 3 and 5

First, let's write the equation in this form (pay attention, the parabola here is going up so it won't be like your graph):

(x-3)(x-5)

For resolve our problem, let's re-write it with a minus before:

-(x-3)(x-5)

You can also expand it and the result will be like this:

-x²+8x-15

O a Olivia went to a dance party. She paid $12 as an entry fee. Once inside the dance party, she paid $2 per snack. She started the night with $30 and spent all her money.

which of the following represents an equation that could be used to model this situation?

A) 2x +12=30
B) 30+2x+12=0
C)2x -12= -30
D)30/2x =12​

Answers

Answer: 2x + 12 = 30

12 is the initial fee, then x is the unknown variable. Since each snack costs 2 dollars, and we don't know how many she had, we can use the equation:

2x +                   12 =               30

|                           |                     |

snack          initial                total

fee                fee                 $ spent

This would be A hope this helps !!

45-[38-{60÷3-(6-9÷3)÷3}]​

Answers

45 - [ 38 - { 60/3 - ( 6 - 3 )/3} ]

45 - [ 38 - { 60/3 - 3/3 } ]

45 - [ 38 - { 20 - 1 } ]

45 - [ 38 - 19 ]

45 - 19

=26

hope this helps....

If f(x) = -2x - 14, then f'(x) =D
Enter the correct answer.

Answers

Answer:

f⁻¹(x) = 1/2x + 7

Step-by-step explanation:

To find the inverse of a function, simply switch x and y and solve for y once more.

f(x) = -2x - 14

x = -2y - 14

x + 14 = -2y

(x + 14)/2 = f⁻¹(x)

1/2x + 7 = f⁻¹(x)

1/2x + 7 is the answer

what is the inverse of the function f(x) = 4x^3 - 9

Answers

Answer:

[tex]f(x)^-^1=\sqrt[3]{\frac{x+9}{4}}[/tex]

Step-by-step explanation:

Finding the inverse of a function is essentially doing a given function in backwards order. An easy trick to doing such is to treat the evaluation (f(x)) like a variable. Think of the function as an equation, then solve the function for (x) in terms of (f(x)).

[tex]f(x)=4x^3-9[/tex]

Inverse operations,

[tex]f(x)=4x^3-9\\\\(f(x))+9=4x^3\\\\\frac{((f(x))+9)}{4}=x^3\\\\\sqrt[3]{\frac{(f(x))+9}{4}}=x[/tex]

Now put this in the form of an inverse function, switch the places of the terms (x) and (f(x)), remember to indicate that it is an inverse function,

[tex]f(x)^-^1=\sqrt[3]{\frac{x+9}{4}}[/tex]

evaluate 9^-1/2
help

Answers

Answer:

1 /3

Step-by-step explanation:

We know that a^-b = 1/a^b

9^-1/2 = 1/9^1/2

The sqrt (9) =3

1 /3

If a time t was 27 seconds correct to the nearest second, determine:
(a) The lower bound.
(b) The upper bound.
(c) The error interval.

Answers

answer:

The answer is (b) The upper bound.

Step-by-step explanation:

Sana makatolong


Which expression represents the second partial sum for Infinity Σ 3(1/2) HURRY PLEASE , I need help on a couple of them

Answers

Answer:

Step-by-step explanation:

The given expression has the first term n=1, so exponent = n-1 = 0

Similarly, for the second term n = 2, n-1 = 1

These exponents eliminate options 2 and 4.

Furthermore, the base is a fraction of 1/2, which applies to options 1 and 2 only.

The only viable option left that satisfies all conditions is option 1, which has exponents 0 and 1, with a base of (1/2).

The second partial sum for [tex]\sum\limit^\infty_{n=1}\; 3* (\frac{1}{2})^{n-1}[/tex] is [tex]3* (\frac{1}{2})^0 + 3* (\frac{1}{2})^1[/tex]

The correct option is (1)

What is Summation ?

A summation, also called a sum, is the result of arithmetically adding numbers or quantities. A summation always contains a whole number of terms.

The given summation is:

[tex]\sum\limit^\infty_{n=1}\; 3* (\frac{1}{2})^{n-1}[/tex]

If we put two values of n i.e., n=1,2. hen we get the following summation series

=[tex]3* (\frac{1}{2})^{1-1} + 3* (\frac{1}{2})^{2-1}[/tex]

=[tex]3* (\frac{1}{2})^0 + 3* (\frac{1}{2})^1[/tex]

The option answer matches with the option 1.

Hence the correct option is (1)

Learn more about summation here:

https://brainly.com/question/17104120

#SPJ2

Scott needs to get a vehicle. He decides to buy a new Ford F250. NYE Auto sells Scott the truck for $36,900. a. NYE Auto charges a destination fee of $450. They also charge $550 for a license fee. If he is also charged 8.25% of the cost of the truck for sales tax, what is the total cost of the truck to Scott?

Answers

Answer:

Car price= $36900

Sales tax 8.25%= $36900 x 8.25% = $3,044.25

Total fees= $1000

Cost of car= 36900+ 3044.25 +1000 = $40,944.25

There's a piece of a pyramid is a rectangle with a width of 4.6 cm and the length of the 9 cm. What is the height in centimeters of the pyramid if it's volume is 82.8 cm^3

Answers

Answer:

6cm

Step-by-step explanation:

Given data

Width= 4.6cm

Length= 9cm

Volume= 82.8cm^3

The expression for the volume is given as

V= lwh/3

82.8= 9*4.6*h/3

cross multiply

82.8*3= 9*4.6*h

248.4= 41.4h

h= 248.4/41.4

h= 6cm

4 5/12 + 3 8/14

ASAP pls and thank you!

Answers

Answer:

671/84 is the answer

Step-by-step explanation:

∠A and \angle B∠B are vertical angles. If m\angle A=(7x-8)

and m\angle B=(6x+17)

, then find the value of x.

Answers

Answer:

x=25

Step-by-step explanation:

We know vertical angles are equal

<A = <B

7x-8 = 6x+17

Subtract 6x from each side

7x-6x-8 = 6x-6x+17

x-8 = 17

Add 8 to each side

x-8+8 = 17+8

x=25

To get a shade of blue, a painter mixes blue paint and white paint in the ratio of 4:1.

How many litres of blue paint and white paint would be needed to get 60 litres of the required

shade of blue?

Which one is correct
55 blue and 5 white
48 blue and 12 white
45 blue and 15 white
40 blue and 20 white

Answers

Answer:

48 blue and 12 white

Explanation:

60 ÷ 5 = 12

4 × 12 = 48

12L white

48L blue

To check:

48 + 12 = 60

48 liters of blue paint and 12 liters of white paint would be needed to get 60 liters of the required shade of blue.

What is ratio?

"I is the comparison of two quantities of the same kind."

What is an equation?

"It is a mathematical statement that consists of equal symbol between two algebraic expressions."

For given question,

To get a shade of blue, a painter mixes blue paint and white paint in the ratio of 4:1

Let 'x' be the common factor of the ratio.

[tex]\Rightarrow \frac{4}{1} = \frac{4\times x}{1\times x}[/tex]

A painter needed 60 liters of the required shade of blue.

So, we get an equation,

⇒ 4x + 1x = 60

where 4x is the amount of blue paint and 1x is the amount of white paint.

We solve above equation.

⇒ 4x + 1x = 60

⇒ 5x = 60

⇒ x = 12 liters

And the value of 4x would be,

⇒ 4x = 4 × 12

⇒ 4x = 48 liters

Therefore, 48 liters of blue paint and 12 liters of white paint would be needed to get 60 liters of the required shade of blue.

Learn more about ratio, equation here:

https://brainly.com/question/1106484

#SPJ3

One week, Isabelle bought 2 bags of apples and 3 bags of oranges. The next week, she bought 1 bag of apples and 2 bags of oranges. If a represents the cost of each bag of apples and r represents the cost of each bag of oranges, which expression could Isabelle use to calculate the difference between the amounts she spent each week?
A. 3 a minus r
B. 3 a minus 5 r
C. (2 a minus 3 r) minus (a minus 2 r)
D . (2 a + 3 r) minus (a + 2 r)

Answers

Answer:

It should be D.

How many different committees can be formed from 12 teachers and 43 students if the committee consists of 3 teachers and 4 ​students?
The committee of 7 members can be selected in BLANK
different ways.​

Answers

Answer:

[tex] \displaystyle 27150200[/tex]

Step-by-step explanation:

we are two conditions

committees can be formed from 12 teachers and 43 studentsthe committee consists of 3 teachers and 4 students

In choosing a committee, order doesn't matter; in case of teachers we need the number of combinations of 3 people chosen from 12

remember that,

[tex] \displaystyle\binom{n}{r} = \frac{n!}{r!(n - r)!} [/tex]

with the condition we obtain that,

[tex]n = 12[/tex][tex]r = 3[/tex]

therefore substitute:

[tex] \displaystyle\binom{12}{3} = \frac{12!}{3!(12 - 3)!} [/tex]

simplify Parentheses:

[tex] \displaystyle\binom{12}{3} = \frac{12!}{3! \cdot9!} [/tex]

rewrite:

[tex] \rm \displaystyle\binom{12}{3} = \frac{12 \times 11 \times 10 \times 9 \times 8 \times 7 \times 6 \times 5 \times 4 \times 3 \times 2 \times 1}{(1 \times 2 \times 3 )\cdot1 \times 2 \times 3 \times 4 \times 5 \times 6 \times 7 \times 8 \times 9} [/tex]

reduce fraction:

[tex] \rm \displaystyle\binom{12}{3} = \frac{12 \times 11 \times 10}{1 \times 2 \times 3 } [/tex]

rewrite 12 and 10:

[tex] \rm \displaystyle\binom{12}{3} = \frac{3 \times 2 \times 2 \times 11 \times 10}{1 \times 2 \times 3 } [/tex]

reduce fraction:

[tex] \rm \displaystyle\binom{12}{3} = 2 \times 11 \times 10[/tex]

simplify multiplication:

[tex] \rm \displaystyle\binom{12}{3} = 220[/tex]

In case of students we need the number of combinations of 4 students choosen from 43 therefore,

[tex] \displaystyle\binom{43}{4} = \frac{43!}{4!(43 - 4)!} [/tex]

simplify which yields:

[tex] \displaystyle\binom{43}{4} = 123410[/tex]

hence,

The committee of 7 members can be selected in BLANK different ways is

[tex] \displaystyle 123410 \times 220[/tex]

[tex] \displaystyle \boxed{27150200}[/tex]

and we're done!

What is the slope of the line that contains the points (-2,7) and (2,3)?

Answers

The slope of a line that passes through the points (-2,7) and (2,3) is y=-1x+5. The slope is -1

The graph of g(x) shown below has the same shape as the graph f(x)=2^x but it's shifted up 1 unit. What is the equation?

Answers

Answer:

B

Step-by-step explanation:

By adding one to the equation, you are shifting it up one along the y-axis, as the picture depicts.

I hope this helped!

Please help me I need to answer this before tomorrow!

Answers

Answer:

Quadrant IV or D

Step-by-step explanation:

It is Quadrant IV or D, since it is a positive x (meaning it must be 1 or 4) and it has a negative y (so 3 or 4). You then find the common answer which is 4 or IV.

Find the measure of each bolded arc Round to the nearest hundredth

Answers

Answer:

57 + x = 360

-57         -57

X = 303 degrees

Step-by-step explanation:

Full circle equals 360 degrees

Given degree is 57

Unknow value will be replaced as x

Unknown value(x) and given degree added together willl equal 360 degrees in total:

57 + x = 360

please help me (-3)2 + (-2)=​

Answers

(-3)2+(-2)
(-6)+(-2) = -8

Answer:

[tex]\left(-3\right)\cdot \:2+\left(-2\right)\\\left(-3\right)\cdot \:2\\\\=-6\\\=-6+\left(-2\right)\\\\=-6-2\\=-8[/tex]

I hope this is what you're looking for

If the sale price is is $75 and the discount is 20% off, what is the original price?

Answers

let the original price be x.

x-20% of x= 75

= x- 0.20x= 75

= 0.8x= 75

= x= 75/0.8

=x= 93.75

Answer: the original price is $93.75

pls mark me brainliesttt :))

hello, i need to solve for x on this one

Answers

Answer:

x = 12.1532°

General Formulas and Concepts:

Trigonometry

[Right Triangles Only] SOHCAHTOA[Right Triangles Only] sinθ = opposite over hypotenuseInverse Trig

Step-by-step explanation:

Step 1: Define

Identify variables

Angle θ = x

Opposite Leg = 4

Hypotenuse = 19

Step 2: Solve for x

Substitute in variables [sine]:                                                                              sinx = 4/19Inverse trig:                                                                                                       x = sin⁻¹(4/19)Evaluate:                                                                                                           x = 12.1532°
Other Questions
1. How many moles are in the following:a.1.29 x 1024 hydrogen atoms in Hb. 7.36 x 1024 oxygen atomsc. 3.28 x 1023 Na atoms2. How many molecules are in 23.0 moles of oxygen?3. How many moles are in 3.4 x 10molecules of H,SO,?4. How many atoms are in 3.0 mole of Na?5. How many molecules are in 200 moles of 03? for math the answer was cylinder with a radiusof 5 units and a heightof 3 units a cylinder with a radiusof 4 units and a heightof 4 units a cylinder with a radiusof 3 units and a heightof 6 units a cylinder with a radiusof 3 units and a heightof 5 units Any line with no slope is parllel to theA) X-axis B) Y-axis C) Line y=x Use5 ft10 ft30 ftWhat is the volume of the rectangular prism? Enter the answer in the box.ft3 Which is the graph of x - y = 1? ano ang naiambag ni juan ponce enrile sa pagkamit ng demokrasya at kalayaan. State Three causes of service delivery protests within communities. Which blood type was donated by the largest percent of people?Percent of Blood Types DonatedmartinoSpringfieldA-9 05% 9% 19% 20% 25% 30% 35% 40% 46% 55 I will make you the Brainiest if you get it right PLEASE!! Select the correct answer.How did the United States try to stop Iran from funding what US officials believe are terrorist organizations?OAby signing a new defensive treaty with IranOB. by placing sanctions on trade with IranOC.by launching a military invasion of IranODby forming an alliance of nations against Iran List the angles in order from the largest to the smallest. A. 25 B. 18 C.12A. C, B, AB. A, C, BC. C, A, BD. A, B, C a For a certain breed of cat, short tails are dominant (T) and long tails arerecessive (t). The Punnett square below shows a cross between two parents.What is the phenotype ratio for this cross? HELP The movement of tectonic plates in two different locations is shown below:Which statement is most likely true?A) An earthquake may occur in Location A only and a volcanic eruption may occur in Location B only.B) Subduction may occur in Location A and a volcanic eruption mah occur in Location B only.C) Subduction may occur in both locationsD) Earthquakes may occur in both location HELPPP ASAAAAAPPPP TST WILL GIVE FULL CREDIT TO BRAINLIST Help me find the answer please I already know how to do it but we usually use the formula for two bases but this is confusing me because of the third number:(((( Why might an emergency team leader dismiss a proposed emergency planthat is time consuming and outside the scope of what the facility can do?O A. The emergency team leader is responsible for keeping the teamfocused and doesn't have time to provide feedback on the planrather than just dismiss it.O B. The emergency team leader is responsible for keeping the teamfocused on completing a feasible emergency plan.O C. The emergency team lead is responsible for understanding theresources and time availabre and necessary to complete the plan.O D. The emergency team leader is responsible for discouraging teamparticipation because it takes up too much valuable time What is the length of MN Which conversions from scientific notation to standard notation are true? Select the four correct answers label the following parts of dictionary entry Why is it that Oskar Schindler, the German Industrialist, saved fewer Jews than Irena, the social worker, but initially received more acclaim than she? compare two linear relationships below. All of the following are true except